ดูหนึ่งข้อความ
  #36  
Old 10 มีนาคม 2012, 20:06
Metamorphosis's Avatar
Metamorphosis Metamorphosis ไม่อยู่ในระบบ
ลมปราณคุ้มครองร่าง
 
วันที่สมัครสมาชิก: 26 กรกฎาคม 2011
ข้อความ: 312
Metamorphosis is on a distinguished road
Default

ค่าย 1/2554 Inequality

$$\sum_{cyc} \frac{a^3}{a^2+ab+b^2} \geqslant \frac{a+b+c}{3} $$

Am-gm
$a^3+a^3+b^3 \geqslant 3a^2b$ และ $b^3+b^3+a^3 \geqslant 3ab^2$
จะได้ว่า $a^3+b^3 \geqslant a^2b+ab^2$

$3a^3 = 2a^3+a^3 \geqslant 2a^3+a^2b+ab^2-b^3 = 2a^3-a^2b+2a^2b-ab^2+2ab^2-b^3$
$3a^3 \geqslant (2a-b)(a^2+ab+b^2)$

จะได้ว่า $$\sum_{cyc} \frac{a^3}{a^2+b^2+ab} \geqslant \frac{2a-b}{3} = \frac{a+b+c}{3} $$
__________________
Fighting for Eng.CU

10 มีนาคม 2012 20:10 : ข้อความนี้ถูกแก้ไขแล้ว 1 ครั้ง, ครั้งล่าสุดโดยคุณ Metamorphosis
ตอบพร้อมอ้างอิงข้อความนี้